You are on page 1of 7

M17 MAT25-21 HOMEWORK 3 SOLUTIONS

1. To Hand In
..............................................................................................................

Supremums and Infimums.

Exercise 1: Supremum Contained in the Set (Abbott Exercise 1.3.7)

Let F be an ordered field, and let A ⊆ F . Show that if a is an upper bound for A and a ∈ A, then a = sup A.

Proof. We need to show that a satisfies (S1) and (S2).


(S1) By hypothesis, a is an upper bound for A.
(S2) Suppose that u ∈ F is some other upper bound. Then x ≤ u for all x ∈ A. Since a ∈ A, it follows, in
particular, that a ≤ u. Hence a is the least upper bound of A.

..............................................................................................................

The Least Upper Bound Property.

Exercise 2: Relationship between Supremums and Infimums (Abbott Exercise 1.3.3)

Let F be an ordered field with the least upper bound property, and let A ⊆ F be nonempty and bounded
below, and define B = {b ∈ F : b is a lower bound for A}. Show that sup B = inf A.

Proof. First of all, note that A is nonempty, and therefore there exists some a ∈ A, which satisfies b ≤ a for all
b ∈ B, because each b is a lower bound for A. Then B is bounded above by a, hence because F has the least
upper bound property, B has a supremum. Let x = sup B. We just need to show that x satisfies axioms (I1) and
(I2) of the definition of the infimum.
(I1) Given any a ∈ A, b ≤ a for all b ∈ B. Therefore a is an upper bound for B. By (S2), since sup B is the
least upper bound of B, we have that sup B ≤ a. Since a ∈ A was arbitrary, it follows that sup B ≤ a for
all a ∈ A, hence sup B is a lower bound for A.
(I2) Given any other lower bound ` for A, note that by definition of B, ` ∈ B. Therefore ` ≤ sup B. Hence
sup B is the greatest lower bound of A.
Hence x is the infimum of A, as desired. 

Exercise 3: Algebraic Properties of Supremums (Abbott Exercise 1.3.5 and 1.3.6)

Let F be an ordered field with the least upper bound property and let A, B ⊆ F be nonempty subsets
which are bounded above. Let c ∈ F , and define c + A := {c + a : a ∈ A}, cA := {ca : a ∈ A}, and
A + B = {a + b : a ∈ A, b ∈ B}.
(b) Show that if c ≥ 0 then sup(cA) = c sup(A).
(c) Show that sup(A + B) = sup(A) + sup(B).

Proof.
(b) We’ll show that sup(cA) ≤ c sup(A) and that c sup(A) ≤ sup(cA).
1
M17 MAT25-21 HOMEWORK 3 SOLUTIONS 2

• (sup(cA) ≤ c sup(A)). Given any ca ∈ cA, since a ∈ A, we have that a ≤ sup(A). Since c ≥ 0, this
implies that ca ≤ sup(cA). Since ca ∈ cA was arbitrary, it follows that c sup(A) is an upper bound
for cA. Hence by (S2) applied to sup(cA), we have that sup(cA) ≤ c sup(A).
• (c sup(A) ≤ sup(cA)). We consider two cases.
– (c = 0). If c = 0, the cA = {0} and c sup(A) = 0. In this case it is clear that c sup(A) ≤ sup(cA).
– (c > 0). By (S1) applied to cA, we have that for any ca ∈ cA, ca ≤ sup(cA). Dividing both sides
by c > 0, we obtain that a ≤ 1c sup(cA). Since this is true for all a ∈ A, we have that 1c sup(cA)
is an upper bound for A. Therefore by (S2) applied to A, it follows that sup(A) ≤ 1c sup(cA).
Multiplying both sides by c > 0, we obtain that c sup(A) ≤ sup(cA), as desired.
(c) We’ll show that sup(A + B) ≤ sup(A) + sup(B) and that sup(A) + sup(B) ≤ sup(A + B).

• (sup(A + B) ≤ sup(A) + sup(B)). Given any a + b ∈ A + B, we have that a ≤ sup(A) and b ≤ sup(B),
so adding these two inequalities together yields a + b ≤ sup(A) + sup(B). Since this is true for all
a + b ∈ A + B, (S2) applied to A + B yields sup(A + B) ≤ sup(A) + sup(B).
• (sup(A) + sup(B) ≤ sup(A + B)). Given any a ∈ A and b ∈ B, we have a + b ≤ sup(A + B), hence
a ≤ sup(A + B) − b. For any b ∈ B, this is true for all a ∈ A, so we have that sup(A) ≤ sup(A + B) − b.
But this is now true for any b ∈ B, hence we have b ≤ sup(A + B) − sup(A) for all b ∈ B. Hence
sup(B) ≤ sup(A + B) − sup(A), which implies that sup(A) + sup(B) ≤ sup(A + B), as desired.

..............................................................................................................
The Real Numbers: Existence and Preliminary Results.

Exercise 4: The Absolute Value Function (Abbott Example 1.2.5/Exercise 1.2.6)

Define a function, called the absolute value function, as follows.


|·|:R→R

x if x ≥ 0
|x| =
−x if x < 0
Show that the absolute value function satisfies the following properties for all a, b ∈ R.
(a) |ab| = |a||b|.
(b) (Triangle Inequality). |a + b| ≤ |a| + |b|.
(c) (Nonnegativity). For all a ∈ R, |a| ≥ 0.
(d) |a| < b if and only if −b < a < b.
(e) (Reverse Triangle Inequality). ||a| − |b|| ≤ |a − b|.

Proof. (a) Without loss of generality, we consider three cases.


• (a ≥ 0, b ≥ 0). Then ab ≥ 0, so |ab| = ab. Also |a| = a and |b| = b, hence |a||b| = ab.
• (a ≥ 0, b ≤ 0). Then ab ≤ 0, so |ab| = −ab. Also |a| = a and |b| = −b, hence |a||b| = −ab.
• (a ≤ 0, b ≤ 0). Then ab ≥ 0, so |ab| = ab. Also |a| = −a and |b| = −b, hence |a||b| = (−a)(−b) = ab.
(b) Note that for any c ∈ R, c ≤ |c|. Then compute

|a + b|2 = (a + b)2 = a2 + 2ab + b2 ≤ |a|2 + 2|a||b| + |b|2 = (|a| + |b|)2 .


Additionally, given any c, d ∈ R with c, d ≥ 0, if c2 ≤ d2 then c ≤ d, since if d < c, it would follow that
d2< c2 contradicting that c2 ≤ d2 . Hence we have |a + b|, |a| + |b| ≥ 0, and |a + b|2 ≤ (|a| + |b|)2 , which
implies that |a + b| ≤ |a| + |b|, as desired.
(c) If a ≥ 0, the |a| = a ≥ 0. If a < 0, then |a| = −a > 0.
(d) • (⇒). Suppose that |a| < b.
– (a ≥ 0). If a ≥ 0, then −b < 0 ≤ a = |a| < b, as desired.
– (a < 0). If a < 0, then −a > 0 and |a| = −a. Hence a < −a = |a| < b. This implies that a < b,
and that −a < b, which shows that −b < a, so we have −b < a < b, as desired.
• (⇐). Suppose that −b < a < b.
M17 MAT25-21 HOMEWORK 3 SOLUTIONS 3

– (a ≥ 0). If a ≥ 0, then |a| = a < b, as desired.


– (a < 0). If a < 0, then since −b < a implies −a < b, we have |a| = −a < b as desired.
(e) Compute

|a| = |a − b + b| ≤ |a − b| + |b|
=⇒ |a| − |b| ≤ |a − b|,
and

|b| = |b − a + a| ≤ |a − b| + |a|
=⇒ |b| − |a| ≤ |a − b|
=⇒ −|a − b| ≤ |a| − |b|.
Putting these together, and using part (d), we obtain that

−|a − b| ≤ |a| − |b| ≤ |a − b|


=⇒ ||a| − |b|| ≤ |a − b|.


..............................................................................................................

Consequences of Completeness of R.

Exercise 5: Intersection of Open Intervals (Abbott Exercise 1.4.3)

Consider, for each n ∈ N, the open intervals (0, 1/n) ⊆ R. Prove that ∞
T
n=1 (0, 1/n) = ∅. Notice that this
demonstrates that the intervals in the Nested Interval Property must be closed for the conclusion of the
theorem to hold.

Proof. Suppose, for contradiction, that x ∈ ∞


T
n=1 (0, 1/n). Then x ∈ (0, 1/n) for all n ∈ N, and hence 0 < x < 1/n
for all n ∈ N. However by the Archimedean property T of R, there exists some m ∈ N satisfying 0 < 1/m < x.
Hence we obtain a contradiction, which implies that ∞ n=1 (0, 1/n) = ∅. 

..............................................................................................................

Existence of Square Roots in R.

Exercise 6: Existence of Square Roots in R


(a) Modify the proof of the existence of 2 in R slightly to show that for any a ∈ R with a ≥ 0, there
exists b ∈ R such that b2 = a. This is often stated as the existence of square roots in R.
(b) Show that if b2 = a, then (−b)2 = a.
(c) Show that for a, b, c ∈ R, if b2 = a and c2 = a, then either b = c or b = −c.
(d) Show that for any a ∈ R with a ≥ 0, there always exists a unique non-negative real number b such
that b2 = a. √ √
(e) Define a function · : R≥0 → R by letting a be the unique non-negative real number b with b2 = a,
whose existence is guaranteed by part (d). √
√ √
(f) Prove that · is strictly increasing, meaning if 0 ≤ a < b, then a < b.


Proof. (a) In this case, we’ll modify the proof of the existence of 2 slightly by considering four cases.
M17 MAT25-21 HOMEWORK 3 SOLUTIONS 4

• (a > 1). Consider the set A = {z ∈ R : z 2 ≤ a}. Note that a is an upper bound for A. Because R
has the least upper bound property, A has a supremum,
√ which we will denote by b. We want to show
that b2 = a. As in the proof of the existence of 2, we make the following definition and calculation.
b2 − a
(1) y =b− ,
b+a
and note that we can write
a(a − 1)(b2 − a)
(2) y2 − a = .
(b + a)2
By axiom (O1) of the definition of a total order, exactly one of the following is true: b2 < a, b2 = a,
b2 > a. We will show that b2 < a and b2 > a cannot happen, and therefore it must be that b2 = a.
– (b2 < a). If b2 < a, then b2 − a < 0, hence Equation 1 implies that y > b. On the other hand
since a > 1, Equation 2 implies that y 2 − a < 0, and therefore that y 2 < a. Hence there exists
a real number y which satisfies y 2 < a, and therefore y ∈ A, yet, b < y. This contradicts that b
is an upper bound for A, which is axiom (S1) of the definition of the supremum.
– (b2 > a). If b2 > a, then b2 − a > 0, and Equation 1 implies that y < b, and since a > 1,
Equation 2 implies that y 2 − a > 0, and therefore y 2 > a. Hence y is an upper bound for A,
and yet y < b, which contradicts axiom (S2) of the definition of the supremum.
Since b2 6< a and b2 6> a, it follows that b2 = a, as desired.
Note, since a > 1, it follows that 12 = 1 < a, and therefore 1 ∈ a. Since b = sup(A), it follows that
b ≥ 1.
• (0 < a < 1). In this case, 1 < a1 , hence we may apply part (a) to obtain b ∈ R, b ≥ 1 with b2 = a1 .
2
Noting that b 6= 0 and rearranging this, we obtain that a = b12 = 1b . Therefore we have obtained
1 1 1 2

b ∈ R, satisfying 0 < b ≤ 1 with b = a, as desired.
• (a = 1). We may take b = 1.
• (a = 0). We may take b = 0.
(b) We proved in class that in a field (−x)(−x) = (x)(x), hence (−b)2 = b2 = a.
(c) Suppose that b 6= c. Without loss of generality, we consider three cases.
• (0 ≤ b < c). Squaring b and c yields 0 ≤ b2 < c2 , contradicting that b2 = c2 .
• (b < c ≤ 0).Squaring b and c yields 0 ≤ c2 < b2 , contradicting that b2 = c2 .
• (b < 0 < c). If b < 0, then −b < 0, hence the previous two cases imply that we must have −b = c.
(d) Given any a ≥ 0, we showed in part (a) that there existed some b ∈ R with b2 = a. If b < 0, part (b)
shows that (−b)2 = a, and hence in either case we have some non-negative real number whose square is a.
If b and c are two non-negative real numbers whose squares are equal to a, then the case (0 ≤ b < c) of
the proof of part (c) shows that we must have b = c. Hence this non-negative square root of a is unique.
(e) Defined! √

(f) Note
√ that by definition ( a)2 = a. Therefore suppose for contradiction that√ 0 ≤ a < b, and yet 0 ≤ b ≤
a. Squaring both sides yields b ≤ a, contradicting that a < b. Therefore · is strictly increasing.


..............................................................................................................

The Limit of a Sequence.

Exercise 7: Equivalence of Definitions of Convergence of a Sequence

Given a sequence (an ) of real numbers and a point a ∈ R, show that the following are equivalent.
(a) (Convergence of a Sequence). For every  > 0, there exists an N ∈ N, such that whenever n ≥ N ,
it follows that |an − a| < .
(b) (Convergence of a Sequence: Topological Version). Given any -neighborhood V (a) of a, there exists
a point in the sequence after which all of the terms are in V (a).
(c) Every -neighborhood of a contains all but a finite number of the terms of (an ).
M17 MAT25-21 HOMEWORK 3 SOLUTIONS 5

Proof. • ((a) =⇒ (b)). Let V (a) be an arbitrary -neighborhood of a. Then by (a), there exists N ∈ N
such that for all n ≥ N , |an − a| < . Then aN is a point in the sequence after which |an − a| < , which
implies that an ∈ V (a).
• ( (b) =⇒ (c)). Let V (a) be an arbitrary -neighborhood of a. By part (b), there exists a point, which
we’ll label as aN , in the sequence after which all of the terms of (an ) are in V (a). Therefore for n ≥ N ,
an ∈ V (a), hence the set of points in the sequence which are not in V (a) is a subset of {a1 , a2 , . . . , aN −1 },
and is therefore a finite set. Therefore all but a finite number of terms of the sequence are in V (a).
• ((c) =⇒ (a)). Let  > 0 be arbitrary. Then by (c), all but a finite number of terms of the sequence are in
V (a), so there is some point aN in the sequence such that if n ≥ N , then an ∈ V (a), which implies that
|an − a| <  for all n ≥ N .


Exercise 8: Uniqueness of Limits (Abbott Theorem 2.2.7/Exercise 2.2.6)

Prove that the limit of a sequence, when it exists, is unique. I.e. if (an ) converges to a and also converges
to b, then a = b.

Proof. Suppose, for contradiction, that (an ) → a and (an ) → b, but a 6= b. Without loss of generality, we can
assume that a < b. Then let  = b−a 4 > 0. Since (an ) → a, there exists N1 ∈ N such that if n ≥ N1 , it follows
that |an − a| < . Similarly, since (an ) → b, there exists N2 ∈ N such that if n ≥ N2 , then |an − b| < . Let
N = max(N1 , N2 ). If n ≥ N , then n ≥ N1 , N2 , hence we compute

b−a
b − a ≤ |a − b| ≤ |a − an + an − b| ≤ |an − a| + |an − b| < ,
2
which is a contradiction, since b > a. Therefore a = b. 

Exercise 9: Examples of Limits of Sequences (Abbott Exercise 2.2.2)

Verify, using the definition of convergence of a sequence, that the following sequences converge to the
proposed limit.
2n+1
(a) limn→∞ 5n+4 = 25 .
2n2
(b) limn→∞ n3 +3
= 0.

Proof. (a) Given  > 0 arbitrary, by the Archimedean property of R, we may choose N ∈ N such that
N > 3−20 3−20 3
25 . Then if n ≥ N , we have n > 25 , which implies that 25n+20 < . Hence for n ≥ N , we have


2n + 1 2 5(2n + 1) − 2(5n + 4)
5n + 4 − 5 =

5(5n + 4)

−3
=
25n + 20
3
= < .
25n + 20
(b) Given  > 0 arbitrary, by the Archimedean property of R, we may choose N ∈ N such that N > 2 . Then
if n ≥ N , we have that n > 2 and hence that n2 < . Then compute

2n2 2 2

= 2n ≤ 2n = 2 < .
n3 + 3 n3 + 3 n3 n

M17 MAT25-21 HOMEWORK 3 SOLUTIONS 6

Exercise 10: Abbott Exercise 2.2.4

Give an example of each of state that the request is impossible. For any that are impossible, prove it.
(a) A sequence with an infinite number of 1’s that does not converge to 1.
(b) A sequence with an infinite number of 1’s that converges to a limit not equal to 1.
(c) A divergent sequence such that for every n ∈ N, it is possible to find n consecutive 1’s somewhere
in the sequence.

Proof. (a) The sequence (1, 2, 1, 3, 1, 4, 1, 5, . . .) has infinitely many 1’s, but is not bounded, and therefore
cannot converge to 1.
(b) This is not possible. Suppose that (an ) had infinitely many 1’s, but converged to a 6= 1. Assume that
1 < a (the other case is handled similarly). Then let  = a − 1 > 0. Since (an ) converges to a, there exists
some N ∈ N such that if n ≥ N , then |an − a| < a − 1. This implies, in particular, that −a + 1 < an − a,
which implies that 1 < an for all n ≥ N . But this means that all but a finite number of the an ’s are strictly
greater than 1, a contradiction.
(c) The sequence (1, 2, 1, 1, 3, 1, 1, 1, 4, 1, 1, 1, 1, 5, . . .) has n consecutive 1’s for any n, but is unbounded, and
therefore divergent.


..............................................................................................................
..............................................................................................................

2. Practice Problems (Don’t hand these in)


The Least Upper Bound Property.

Exercise 11: Supremums of Unions (Abbott Exercise 1.3.4)

Let F be an ordered field with the least upper bound property and let A1 , A2 , . . . Ak , . . . be a collection of
nonempty subsets of F , each of which is bounded above.
(a) Find a formula for sup(A1 ∪ A2 ). S
(b) Extend this to a formula for sup ( nk=1 ). S
(c) Further extend this to a formula for sup ( ∞k=1 ).

..............................................................................................................

The Real Numbers: Existence and Preliminary Results.

Exercise 12: “∀ ” Proofs (Abbott Exercise 1.2.10)

Decide which of the following are true statements. If they are true, prove them, and if they are false, find
a specific counterexample.
(a) Two real numbers satisfy a < b if and only if a < b +  for all  > 0.
(b) Two real numbers satisfy a < b if a < b +  for all  > 0.
(c) Two real numbers satisfy a ≤ b if and only if a < b +  for all  > 0.

Exercise 13: Finding Supremums

Let a < b be real numbers. Find the supremums of the following sets, and prove your results.
(c) [a, b) = {r ∈ R | a ≤ r < b}.
(d) (a, b] = {r ∈ R | a < r ≤ b}.

..............................................................................................................
M17 MAT25-21 HOMEWORK 3 SOLUTIONS 7

The Limit of a Sequence.

Exercise 14: Convergence of a Constant Sequence

Let x ∈ R be a real number, and define a sequence (an ) with an = x for all n ∈ N. Show that (an ) converges
to x.

..............................................................................................................
Divergence.

Exercise 15: Divergence

Show that the sequence (n)∞


n=1 = (1, 2, 3, 4, . . .) diverges.

..............................................................................................................
Bounded Sequences.

Exercise 16: Bounded Sequence

Recall the definition of a bounded subset of an ordered field (??). Show that a sequence (xn ) of real numbers
is bounded if and only if the set of terms of (xn ), i.e. {xn : n ∈ N}, is a bounded subset of R.

Exercise 17: Bounded Sequences

Is it true that every bounded sequence is convergent? Prove or find a counterexample.

..............................................................................................................

You might also like